Đến nội dung

the unknown nội dung

Có 210 mục bởi the unknown (Tìm giới hạn từ 30-05-2020)



Sắp theo                Sắp xếp  

#646461 $a, b, c > 0.$ Chứng minh: $\sum \frac{a^3}{a^2+...

Đã gửi bởi the unknown on 25-07-2016 - 20:06 trong Bất đẳng thức và cực trị

Bài 2: a, b, c > 0. ab + bc + ca = 1. CMR:

$a^{2}+b^{2}+c^{2}+\frac{8abc}{(a+b)(b+c)(c+a)} \geq 2$

 

Bài 4: a, b, c > 0. CMR:

$\frac{a^{2}+b^{2}+c^{2}}{ab+bc+ca} + \frac{8abc}{(a+b)(b+c)(c+a)}\geq 2$

Về cơ bản hai bài này là giống nhau nên mình gộp chung:

Không mất tính tổng quát giả sử $c$ là số nhỏ nhất.

Ta có:

$\frac{a^2+b^2+c^2}{ab+bc+ca}+\frac{8abc}{(a+b)(b+c)(c+a)}\geq 2$

$\Leftrightarrow M(a-b)^2+N(a-c)(b-c)\geq 0$

Trong đó:

$M=\frac{1}{ab+bc+ca}-\frac{2c}{(a+b)(b+c)(c+a)}$

$N=\frac{1}{ab+bc+ca}-\frac{1}{(b+c)(c+a)}$

Dễ chứng minh được:

$N=\frac{c^2}{(b+c)(c+a)(ab+bc+ca)}\geq 0$

$M=\frac{ab(a+b)+bc(b-c)+ca(a-c)}{(ab+bc+ca)(a+b)(b+c)(c+a)}\geq 0$

Vậy $M,N\geq 0$ nên bất đẳng thức đúng. Vậy ta có điều phải chứng minh. Đẳng thức xảy ra khi $a=b=c$. ( Nếu điều kiện là $a,b,c$ không âm thì bài toán còn xảy ra đẳng thức tại $a=b$ và $c=0$ nữa)




#646426 Tìm $\text{max S(a+b+c)}$

Đã gửi bởi the unknown on 25-07-2016 - 15:15 trong Số học

Với số nguyên dương $n$, kí hiệu $S(n)$ là tổng các chữ số viết trong hệ thập phân của $n$. Giả sử có ba số nguyên dương $a,b,c$ sao cho $S(a+b)<5$, $S(b+c)<5$, $S(c+a)<5$. Tìm giá trị lớn nhất của $S(a+b+c)$.




#646398 Tìm tất cả các số nguyên dương giảm.

Đã gửi bởi the unknown on 25-07-2016 - 10:49 trong Số học

Một số nguyên dương $a$ được gọi là giảm về số nguyên dương $b$ nếu ta chia $a$ cho chữ số hàng đơn vị của nó thì ta được số $b$. ( Ví dụ như $2016$ giảm về $\frac{2016}{6}=336$)

Hãy tìm tất cả các số nguyên dương có thể giảm về số $1$ sau một số lần giảm. Ví dụ như $12$ là một số có tính chất như vậy vì $12$ giảm về $6$ và $6$ giảm về $1$.

(Mexico 2014)

 

P.s




#645972 CMR: $a^2+b^2+c^2 + 2abc + 3 \geq (a + 1)(b + 1)(c + 1)$

Đã gửi bởi the unknown on 22-07-2016 - 13:32 trong Bất đẳng thức và cực trị

Cho a,b,c dương. Chứng minh rằng:

$a^2+b^2+c^2 + 2abc + 3 \geq  (a + 1)(b + 1)(c + 1)$

 

BĐT tương đương: $a^2+b^2+c^2+abc+2\geq ab+bc+ca+a+b+c$

Theo nguyên lí Dirichlet thì trong ba số $a-1,b-1.c-1$ có hai số cùng dấu. Không giảm tổng quát giả sử là $(a-1),(b-1)$. Khi đó $(a-1)(b-1)\geq 0 \Rightarrow ab+1\geq a+b$

$\Rightarrow abc+c\geq ac+bc$

Như vậy ta chỉ cần chứng minh: $a^2+b^2+c^2+2\geq ab+2c+a+b\Leftrightarrow (a-1)^2+(b-1)^2+2(c-1)^2+(a-b)^2\geq 0$ (luôn đúng).

Vậy bất đẳng thức được chứng minh. đẳng thức xảy ra khi và chỉ khi $a=b=c=1$.




#645917 $\sum_{k=1}^n \frac{f(k)}{k^2}...

Đã gửi bởi the unknown on 21-07-2016 - 20:25 trong Tổ hợp và rời rạc

 

Cho $n \in \mathbb{N^*}$ và $f : \{1,2,..,n\} \rightarrow \{1,2..,n\}$ là một song ánh . Chứng minh rằng : 
$\sum_{k=1}^n \frac{f(k)}{k^2} \ge \sum_{k=1}^n \frac{1}{k}$

 

Bài này nếu dùng bất đẳng thức hoán vị thì nhanh rồi nhỉ  :)

Nhắc lại về bất đẳng thức hoán vị: Cho hai dãy số thực $(a_1,a_2,...,a_n)$ và  $(b_1,b_2,...,b_n)$ cùng tăng hoặc cùng giảm. Khi đó giả sử $(b_{k_1},b_{k_2},...,b_{k_n})$ là một hoán vị của $(b_1,b_2,...,b_n)$ thì ta có bất đẳng thức: $a_1b_1+a_2b_2+...+a_nb_n\geq a_1b_{k_1}+a_2b_{k_2}+...+a_nb_{k_n}\geq a_1b_n+a_2b_{n-1}+...+a_nb_1$.

Như vậy nhận xét rằng $(1,2,...,n)$ và $(\frac{1}{n^2},\frac{1}{(n-1)^2},...,1)$ là hai dãy cùng tăng. Hơn nữa ánh xạ $f: \{1,2,...,n\} \rightarrow \{1,2,...,n\}$ là một song ánh nên $(f(1),f(2),...,f(n))$ là một hoán vị của $(1,2,...,n)$. Từ đó theo bất đẳng thức hoán vị ta được: $\sum_{k=1}^n \frac{f(k)}{k^2} \ge \sum_{k=1}^n \frac{1}{k}$.

Vậy bài toán được chứng minh.

 

P.s: Hóng cách giải hay hơn. Chứ dùng bất đẳng thức hoán vị thì ác quá.   :D




#645773 ĐỀ THI OLYMPIC GẶP GỠ TOÁN HỌC LẦN VIII

Đã gửi bởi the unknown on 21-07-2016 - 08:31 trong Thi HSG cấp Tỉnh, Thành phố. Olympic 30-4. Đề thi và kiểm tra đội tuyển các cấp.

Khối 10 bài 3 ( bất đẳng thức): Cho $\Delta ABC$ có độ dài các cạnh không vượt quá $1$. Gọi $p,R,r$ lần lượt là nửa chu vi, bán kính đường tròn ngoại tiếp và nội tiếp tam giác. Chứng minh rằng:

$p(1-2Rr)\leq 1$

 

Kí hiệu $a,b,c$ là các cạnh của tam giác. Ta sử dụng các hệ thức lượng quen thuộc: $S=\frac{abc}{4R},S=pr$. Từ đó ta có:

$p(1-2Rr)=p-2Rpr=p-2RS=p-\frac{abc}{2}=\frac{a+b+c-abc}{2}$.

Do đó bất đẳng thức được chứng minh nếu ta chứng minh được: $a+b+c-abc\leq 2$ với $0<a,b,c\leq 1$ và $a,b,c$ là các cạnh của tam giác.

Do $0<c\leq 1$ và $1-ab\leq 0$ nên $a+b+c-abc=a+b+c(1-ab)\leq a+b+1-ab$

Do đó ta chứng minh $a+b+1-ab\leq 2$ tương đương $(a-1)(b-1)\geq 0$ (luôn đúng do $0<a,b,c\leq 1$)

Vậy bài toán được chứng minh. Đẳng thức xảy ra khi $a=b=c=1$, tức là $\Delta ABC$ đều và có các cạnh bằng $1$.

 

P.s: Bài 2 lớp 10 em có ý tưởng là dùng quy nạp để chứng minh. Tuy nhiên lập luận hơi lỏng lẻo nên chưa dám đăng  :D




#645408 $n=\pm 1^2\pm 2^2\pm \cdots \pm k^2$

Đã gửi bởi the unknown on 18-07-2016 - 15:24 trong Số học

(Olympic GGTH 2016) Chứng minh rằng với mỗi số nguyên $n$ thì tồn tại vô hạn cách để biểu diễn $n$ dưới dạng:

$n=\pm 1^2\pm 2^2\pm \cdots \pm k^2$

với số nguyên dương $k$ và các dấu $+,-$ được chọn phù hợp.




#645367 CMR: Trong dãy có số các số 2 nhiều hơn số các số 1

Đã gửi bởi the unknown on 18-07-2016 - 08:06 trong Toán rời rạc

gọi các số trong dãy là $2^{a}$ với$1\leq a\leq 2005$

xét a lẻ ta có $2^{a}=2^{2k+1} \equiv 2^{2k}.2\equiv 4^{k}.2\equiv 2 (mod 3)$ tổng các chữ số sau cùng của các số này là 2

xét a chẵn thì $2^{2k}\equiv 1 (mod 3)$ tổng các chữ số sau cùng của các chữ số này là 1 
suy ra điều phải chứng minh 
p/s: không biết em làm vậy có chặt chẽ hay không nữa :))

Theo mình nghĩ là thế này: Nếu bạn xét modulo $3$ thì theo mình nghĩ điều đó là chưa đủ, chẳng hạn như nếu một số chia $3$ dư $1$ thì nếu ta thực hiên như đề bài ta sẽ quy về một số có $1$ chữ số chia $3$ dư $1$ như cách của bạn ( phần này đúng không có vấn đề gì  ^_^ ). Nhưng vấn đề là ngoài số $1$ còn các số $4$ và $7$ nữa. Nên nếu xét như bạn thì ta không thể khẳng định số số $2$ nhiều hơn số số $1$. Nếu muốn chắc chắn hơn thì cách tốt hơn là xét modulo $9$ ( dài hơn xíu nhưng cũng không có vấn đề gì  ^_^ )

Mạn phép giải luôn  ^_^ : Dễ chứng minh được $2^a$ chia $9$ dư $1$ khi và chỉ khi $a=6k$ và $2^a$ chia $9$ dư $2$ khi và chỉ khi $a=6k+1$. Do đó bằng cách đếm các số chia hết cho $6$ và chia $6$ dư $1$ ta dễ thấy số các số chia $9$ dư $2$ nhiều hơn số các số chia $9$ dư $1$ nên lập luận tiếp ta suy ra điều phải chứng minh.




#645366 Inequalities From 2016 Mathematical Olympiads

Đã gửi bởi the unknown on 18-07-2016 - 07:51 trong Bất đẳng thức - Cực trị

Bài 49 (Bulgaria). Cho $a,b,c,d0$ là bốn số thực dương. Chứng minh rằng

\[\frac {a+\sqrt{ab}+\sqrt[3]{abc}+\sqrt[4]{abcd}}{4} \leq \sqrt[4]{a \cdot \frac{a+b}{2} \cdot \frac{a+b+c}{3} \cdot \frac{a+b+c+d}{4}}.\]
 

Ta có các BĐT sau theo AM-GM cho 4 số thực dương:

$\frac{1}{4}(1+\frac{2a}{a+b}+\frac{3a}{a+b+c}+\frac{4a}{a+b+c+d})\geq \sqrt[4]{\frac{2a}{a+b}.\frac{3a}{a+b+c}.\frac{4a}{a+b+c+d}}$

 

$\frac{1}{4}\left ( 1+1+\frac{2b}{a+b}+\frac{4c}{a+b+c+d} \right )\geq \sqrt[4]{\frac{2b}{a+b}.\frac{4c}{a+b+c+d}}$

 

$\frac{1}{4}\left ( 1+\frac{2b}{a+b}+\frac{3c}{a+b+c}+\frac{4d}{a+b+c+d}\right )\geq \sqrt[4]{\frac{2b}{a+b}.\frac{3c}{a+b+c}.\frac{4d}{a+b+c+d}}$

 

$\frac{1}{4}(1+\frac{2a}{a+b}+\frac{3b}{a+b+c}+\frac{4b}{a+b+c+d})\geq \sqrt[4]{\frac{2a}{a+b}.\frac{3b}{a+b+c}.\frac{4b}{a+b+c+d}}$

 

Cộng tất cả các bất đẳng thức trên lại và ta thu được:

 

$4\geq \sqrt[4]{\frac{2a}{a+b}.\frac{3a}{a+b+c}.\frac{4a}{a+b+c+d}}+\sqrt[4]{\frac{2b}{a+b}.\frac{4c}{a+b+c+d}}+\sqrt[4]{\frac{2b}{a+b}.\frac{3c}{a+b+c}.\frac{4d}{a+b+c+d}}+\sqrt[4]{\frac{2a}{a+b}.\frac{3b}{a+b+c}.\frac{4b}{a+b+c+d}}$

 

Suy ra:

 

$\sqrt[4]{a.\frac{a+b}{2}.\frac{a+b+c}{3}.\frac{a+b+c+d}{4}}\geq \frac{1}{4}(a+\sqrt{ab}+\sqrt[4]{abc.\frac{a+b+c}{3}}+\sqrt[4]{abcd})\geq \frac{1}{4}(a+\sqrt{ab}+\sqrt[4]{abc.\sqrt[3]{abc}}+\sqrt[4]{abcd})=\frac{a+\sqrt{ab}+\sqrt[3]{abc}+\sqrt[4]{abcd}}{4}$

Vậy bài toán được chứng minh. Đẳng thức xảy ra khi và chỉ khi $a=b=c=d$.

 

Remark




#645310 $\sum (\frac{a}{a+b})^n\geq \fra...

Đã gửi bởi the unknown on 17-07-2016 - 17:41 trong Bất đẳng thức - Cực trị

Chứng minh bất đẳng thức: 

$\sum (\frac{a}{a+b})^n\geq \frac{3}{2^n}$

với $n$ là một số nguyên dương cho trước, $n\geq 2$ và $a,b,c$ dương bất kì.




#645141 Tìm tất cả các cặp $(a,b)$ sao cho $q^{2}+r=1977.$

Đã gửi bởi the unknown on 16-07-2016 - 10:19 trong Số học

(IMO 1977) http://www.artofprob...c6h61028p367399

 

Cho mới lạ, thử đặt lại bài toán là $q^2+r=2017$ thử xem  :D




#644849 Inequalities From 2016 Mathematical Olympiads

Đã gửi bởi the unknown on 13-07-2016 - 21:26 trong Bất đẳng thức - Cực trị

Bài 48 (Japan MO Final). Cho bốn số thực $a,b,c,d$ thỏa mãn điều kiện $ab+bc+cd =1.$
Tìm giá trị nhỏ nhất của biểu thức $$P (a^2+ac+c^2)(b^2+bd+d^2).$$
 

Bài 48: Ta xét: $S=(a^2+ac+c^2)(b^2+bd+d^2)-(ab+bc+cd)^2=(ad-bc)(ab+ad+cd)=(ad-bc)(1+ad-bc)=(ad-bc+\frac{1}{2})^2-\frac{1}{4}\geq -\frac{1}{4}$

$\Rightarrow P\geq (ab+bc+cd)^2-\frac{1}{4}=\frac{3}{4}$

Đẳng thức xảy ra tương đương với việc ta có hệ: $\left\{\begin{matrix} ad-bc=-\frac{1}{2}\\ ab+bc+cd=1\\ \end{matrix}\right.$

Hệ này có nghiệm, ví dụ như bộ nghiệm: $(a,b,c,d)\sim (0,\frac{1}{2},1,\frac{1}{2})$

Như vậy ta có giá trị nhỏ nhất của $P$ là $\frac{3}{4}$, xảy ra chẳng hạn khi $(a,b,c,d)\sim (0,\frac{1}{2},1,\frac{1}{2})$.




#644681 Thảo luận về Đề thi và Lời giải của IMO 2016

Đã gửi bởi the unknown on 12-07-2016 - 16:24 trong Thi HSG Quốc gia và Quốc tế

Cho em hỏi là bài 6 có phải của Việt Nam mình ra không vậy?

Không đâu bạn, nghe bảo là do Geoff Smith, người nước Anh đề xuất.




#644673 $n\mid (2^m-1)^n+1$.

Đã gửi bởi the unknown on 12-07-2016 - 15:58 trong Số học

Tham khảo ở đây: http://diendantoanho...-cho-nmid-15n1/




#644671 Thảo luận về Đề thi và Lời giải của IMO 2016

Đã gửi bởi the unknown on 12-07-2016 - 15:53 trong Thi HSG Quốc gia và Quốc tế

 

attachicon.gifIMO.png

 

 

Bài 3. Cho $P=A_1A_2\ldots A_k$ là một đa giác lồi trong mặt phẳng. Các đỉnh $A_1,A_2,\ldots A_k$ có tọa độ là các số nguyên và nằm trên một đường tròn. Gọi $S$ là diện tích của $P$. Một số tự nhiên $n$ lẻ thỏa mãn bình phương độ dài các cạnh của $P$ đều chia hết cho $n$. Chứng minh rằng $2S$ là một số tự nhiên chia hết cho $n$

 

Đây là một lời giải em tham khảo ở AoPS cho bài số 3, bài được cho là khó nhất của IMO năm nay  :)

Bài 3: Trước hết, giả sử rằng đa giác $A_1A_2\ldots A_k$ có một đường chéo thỏa mãn bình phương của đường chéo đó là một số nguyên chia hết cho $n$, khi đó ta sẽ cắt đa giác này bằng đường chéo này thành hai đa giác với số cạnh nhỏ hơn. Do đó không giảm tổng quát có thể giả sử đa giác $A_1A_2\ldots A_k$ không có một đường chéo nào có bình phương chia hết cho $n$.

 

Giả sử $k\ge 4$. Không mất tính tổng quát, ta sẽ chứng minh cho trường hợp $n=p^a$ ($p$ là số nguyên tố lẻ). Khi đó, xét một số tự nhiên $r$ thỏa $a>r\geq 0$ và $p^r\mid A_iA_j^2$ với mọi $i\neq j$, $0<i,j\le k$. Hiển nhiên ta có $r+1\leq a$ nên $p^{r+1}\mid p^a\mid A_iA_j^2$ nếu $A_i,A_j$ là hai đỉnh kề nhau.

 

Ta sẽ chứng minh bằng quy nạp rằng nếu $r$ là một số tự nhiên thỏa mãn điều kiện trên thì $r+1$ cũng sẽ thỏa mãn các điều kiện trên. Do tất cả các đỉnh của đa giác cùng nằm trên một đường tròn nên xét cụ thể cho một tứ giác $A_{i-1}A_iA_{i+1}A_j$ và cho đơn giản, ta đặt: $A_{i-1}A_i=b,A_{i}A_{i+1}=c,A_{i+1}A_j=d,A_jA_{i-1}=e,A_{i-1}A_{i+1}=f,A_iA_j=g$.

 

Và theo giả thiết quy nạp, ta đặt: $b^2=p^ax,c^2=p^ay,d^2=p^rz,e^2=p^rk,f^2=p^rw,g^2=p^rt$ ( với $x,y,z,k,w,t$ là các số nguyên) Khi đó theo định lý Ptolemy thì ta có được:$bd+ce=fg \Rightarrow f^2g^2=b^2d^2+c^2e^2+2bcde=p^{a+r}(xz+yk+2\sqrt{xyzk})$. Từ đó ta có $\sqrt{xyzk}$ là một số hữu tỉ nên cũng là một số nguyên. Do đó ta có được: $p^{a+r}\mid f^2g^2\Rightarrow v_{p}(f^2g^2)\geq a+r$. Tuy nhiên: vì $A_iA_j$ và $A_{i-1}A_{i+1}$ là các đường chéo nên $v_{p}(f^2)\leq a-1\Rightarrow v_{p}(g^2)\geq r+1\Rightarrow p^{r+1}\mid g^2$

 

Tức là khi đó $p^{r+1}\mid A_iA_j^2$ với $A_iA_j$ là một đường chéo và tương tự với các đường chéo khác, ta có được $p^{r+1}\mid A_iA_j^2$ với mọi $i\neq j$, $0<i,j\leq k$. Như vậy theo nguyên lí quy nạp, giả thiết được chứng minh. Khi đó theo giả thiết thì $p^{a-1}\mid A_iA_j^2$ với mọi $i\neq j$ và cũng chứng minh tương tự mà $p^a\mid A_iA_j^2$ với mọi $i\neq j$. Đây là điều vô lí ( do ta đã giả sử rằng không có đường chéo nào chia hết cho $p^a$).

 

Do đó $k<4$, tức là ta sẽ quy về trường hợp đơn giản $k=3$. Ta có thể chứng minh điều này cách dễ dàng vì các đỉnh $A_1,A_2,A_3$ có tọa độ nguyên nên dễ dàng chứng minh được $2S$ là số nguyên.

Hơn nữa theo công thức Hê-rông thì $2S=\frac{\sqrt{4a^2b^2-(a^2+b^2-c^2)^2}}{2}$ ($a,b,c$ là các cạnh của tam giác) nên $4.(2S)^2=(4a^2b^2-(a^2+b^2-c^2)^2)\vdots n^2$. Do $n$ lẻ nên $(4,n)=1$ do đó $n^2\mid (2S)^2$ nên $n\mid 2S$.

Vậy bài toán được chứng minh hoàn toàn.$\blacksquare$

 

P.s: Hình như ngày 2 không có hình  :(




#644636 Chứng minh $(a^{2}+b^{2}+c^{2})(\frac...

Đã gửi bởi the unknown on 12-07-2016 - 13:12 trong Bất đẳng thức và cực trị

Cho a, b, c là độ dài một tam giác không nhọn. Chứng minh rằng:

$(a^{2}+b^{2}+c^{2})(\frac{1}{a^{2}}+\frac{1}{b^{2}}+\frac{1}{c^{2}})\geq 10$

Có thể giả sử $\widehat{A}\geq 90\Rightarrow b^2+c^2\leq a^2$. Ta đặt $b^2+c^2=x$ và $a^2=y$ thì $x\leq y$

Ta có: $(a^2+b^2+c^2)(\frac{1}{a^2}+\frac{1}{b^2}+\frac{1}{c^2})\geq (a^2+b^2+c^2)(\frac{1}{a^2}+\frac{4}{b^2+c^2})=(x+y)(\frac{4}{x}+\frac{1}{y})=5+\frac{4y}{x}+\frac{x}{y}=5+\frac{3y}{x}+(\frac{x}{y}+\frac{y}{x})\geq 10$.

Đẳng thức xảy ra khi và chỉ khi $a^2=b^2+c^2$ và $b=c$ , tức là tam giác đó là tam giác vuông cân. $\blacksquare$




#644464 $S<\sqrt[3]{4}$

Đã gửi bởi the unknown on 11-07-2016 - 09:41 trong Bất đẳng thức và cực trị

Cho $a$ là một số thực thỏa mãn $a^5-a^3+a-2=0$. Chứng minh rằng ta có:

$S= \frac{a^{16}+a^{12}+7a^8+12a^4+12}{a^{12}+7a^8+7a^4+12}<\sqrt[3]{4}$

Spoiler




#644460 MAX: $T=\frac{a(b+c)}{(b+c)^2+a^2}+\frac...

Đã gửi bởi the unknown on 11-07-2016 - 09:14 trong Bất đẳng thức và cực trị

Cho a,b,c dương. Tìm GTLN của:

$T=\frac{a(b+c)}{(b+c)^2+a^2}+\frac{b(c+a)}{(c+a)^2+b^2}+\frac{c(a+b)}{(a+b)^2+c^2}$

Chuẩn hóa $a+b+c=3$. Khi đó ta sẽ tìm GTLN của: $\sum \frac{a(3-a)}{(3-a)^2+a^2}$.

Ta sẽ chứng minh: $\frac{a(3-a)}{(3-a)^2+a^2}\leq \frac{9}{25}a+\frac{1}{25}\Leftrightarrow \frac{9(a-1)^2(2a+1)}{25(a^2+(3-a)^2)}\geq 0$ ( hiển nhiên đúng)

Do đó $\sum \frac{a(3-a)}{a^2+(3-a)^2}\leq \sum (\frac{9}{25}a+\frac{1}{25})= \frac{6}{5}$

Nên GTLN của $T$ là  $\frac{6}{5}$, xảy ra khi $a=b=c$.




#644400 Tìm $x,y$ nguyên dương để $x^2y^4-y^3+1$ là số chính phương

Đã gửi bởi the unknown on 10-07-2016 - 21:01 trong Số học

Tìm $x,y$ nguyên dương để $x^2y^4-y^3+1$ là số chính phương

Đặt $S=x^2y^4-y^3+1$.

Trước hết với trường hợp $y=1$ thì $S=x^2$ là số chính phương với $x$ tùy ý. Do đó cặp số $(x,y)$ thỏa là $(k,1)$ với $k$ là số nguyên dương tùy ý.

Xét $y\geq 2$. Khi đó ta cần có bổ đề sau:

Bổ đề: Với $k$ là số nguyên dương, $k\geq 2$ thì khi đó, nếu $a,b$ là các số nguyên dương thỏa:

$\left\{\begin{matrix} ab=k^3-1\\ k^2\mid a+b\\ \end{matrix}\right.$

thì $(a,b)$ là một hoán vị của $(k^3-1;1)$.

Chứng minh

Trở lại bài toán: Đặt $S=t^2$ suy ra $(xy^2-t)(xy^2+t)=y^3-1$. Nếu đặt $a=xy^2-t$ và $b=xy^2+t$ thì $ab=y^3-1$ và $ y^2\mid a+b$ nên theo bổ đề ta có được $xy^2+t=y^3-1$ và $xy^2-t=1$. Suy ra $2xy^2=y^3\Rightarrow 2x=y$. Suy ra phương trình có nghiệm là $(k,2k)$ với $k$ là số nguyên dương tùy ý, thử lại thấy thỏa.

Kết luận: Phương trình có các nghiệm nguyên là $(k;1)$ và $(k;2k)$ với $k$ nguyên dương tùy ý.




#644370 Giải phương trình nghiệm nguyên dương sau:$y^3 -3^x =100.$

Đã gửi bởi the unknown on 10-07-2016 - 17:05 trong Số học

Giải phương trình nghiệm nguyên dương sau: $$y^3 -3^x =100.$$

Dễ chứng minh được $y\equiv 1 (\text{mod 3})$ và đặt $y=3k+1$. Từ đó biến đổi ta có: $3k^3+3k^2-3^{x-2}=11-k$.

Đến đây nếu ta đã loại được giá trị $x=2$ thì để ý rằng $11-k$ chia hết cho $3$. Từ đó ta chặn giá trị $k$ và tìm được bộ số duy nhất thỏa là $(5;7)$.




#643772 Tính $j-i$

Đã gửi bởi the unknown on 05-07-2016 - 20:23 trong Số học

Số $a_n$ ($n\geq 1$) được lập bằng cách viết liền các số $1,2,3,...,n$ ( ví dụ $a_{12}=123456789101112$). Giả sử $i,j$ là hai số nhỏ nhất ($i< j$) sao cho $11\mid a_i$ và $11\mid a_j$. Tính $j-i$.




#643644 USA TSTST 2016

Đã gửi bởi the unknown on 04-07-2016 - 17:41 trong Thi HSG Quốc gia và Quốc tế

USA TSTST 2016

(USA Team Selection Test for the Selection Team 2016)

 

Ngày 1 (25/6/2016)

 

Bài 1: Cho hai đa thức $A=A(x,y)$ và $B=B(x,y)$ là hai đa thức hai biến với hệ số thực. Giả sử rằng $A(x,y)/B(x,y)$ là đa thức theo $x$ với vô số giá trị của $y$ và là đa thức theo $y$ với vô số giá trị của $x$. Chứng minh rằng $B | A$, tức là tồn tại một đa thức $C$ với hệ số thực thỏa $A=B.C$

 

Bài 2: Cho $ABC$ là một tam giác không đều với trực tâm $H$ và tâm ngoại tiếp $O$. Gọi $M,N$ là trung điểm của $AH$ và $BC$ tương ứng. Giả sử đường tròn $\gamma$ với đường kính $AH$ cắt $(ABC)$ tại $G\neq A$ và cắt đường thẳng $AN$ tại $Q\neq A$. Tiếp tuyến tại $G$ của $\gamma$ cắt đường thẳng $OM$ tại $P$. Chứng minh rằng giao điểm của hai đường tròn $(GNQ)$ và $(MBC)$ cắt nhau tại điểm $T$ nằm trên $PN$.

 

Bài 3: Có tồn tại hay không một đa thức khác hằng $Q(n)$ với hệ số nguyên thỏa mãn tính chất: Với mọi số nguyên dương $n>2$ thì các số:

$Q(0),Q(1),Q(2),...,Q(n-1)$

nhận không quá $0,499n$ số dư phân biệt theo modulo $n$.

 

Ngày 2 (27/6/2016)

Bài 4: Giả sử $n$ và $k$ là các số nguyên dương thỏa mãn:

$\underbrace{\varphi (\varphi (...\varphi }_{\text{k lần}}(n)...))=1$

Chứng minh rằng $n\leq 3^k$.

Trong đó kí hiệu $\varphi(n)$ là số các số nguyên dương trong tập $\left \{ 1,2,3,...,n \right \}$ mà nguyên tố cùng nhau với $n$.

 

Bài 5: Trong mặt phẳng tọa độ có hữu hạn các bức tường là các đoạn thẳng không giao nhau và không có đoạn nào song song với các trục tọa độ. Một chiếc xe ủi bắt đầu từ một điểm bất kì trên mặt phẳng và di chuyển theo hướng dương của trục hoành. Mỗi khi tông một bức tường, chiếc xe sẽ rẽ một góc vuông và đi xa ra khỏi bức tường đó. ( Vì vậy mà chiếc xe luôn chuyển động song song với các trục tọa độ).

Chứng minh rằng chiếc xe không thể tông cả hai mặt của tất cả các bức tường.

 

Bài 6: Một tam giác $ABC$ có tâm nội tiếp $I$ và đường tròn nội tiếp tiếp xúc với các cạnh $BC,CA,AB$ lần lượt tại các điểm $D,E,F$. Gọi $K$ là chân đường vuông góc hạ từ $D$ xuống $EF$. Giả sử rằng đường tròn ngoại tiếp tam giác $AIB$ gặp đường tròn nội tiếp tại hai điểm phân biệt $C_1,C_2$, và đường tròn ngoại tiếp tam giác $AIC$ gặp đường tròn nội tiếp tại hai điểm phân biệt $B_1,B_2$. Chứng minh rằng trục đẳng phương của $(BB_1B_2)$ và $(CC_1C_2)$ đi qua trung điểm $M$ của $DK$.

 

Nguồn

P.s




#643602 Kĩ thuật đổi biến trong chứng minh bất đẳng thức

Đã gửi bởi the unknown on 04-07-2016 - 11:01 trong Chuyên đề toán THCS

Thấy Topic khá hay mà đóng băng cũng gần một tháng rồi nên hơi buồn  :( . Mình xin đóng góp một số bài bất đẳng thức mà kĩ thuật đổi biến là một điều khá cần thiết:

Bài 5: Cho các số thực $a,b,c$ đôi một khác nhau và $k$ là số thực bất kì. Chứng minh:

$\left ( \frac{k+a}{b-c} \right )^2+\left ( \frac{k+b}{c-a} \right )^2+\left ( \frac{k+c}{a-b} \right )^2\geq 2$

Bài 6: Cho $a,b,c$ là các số thực bất kỳ. Chứng minh rằng:

$\left ( \frac{a}{a-kb} \right )^2+\left ( \frac{b}{b-kc} \right )^2+\left ( \frac{c}{c-ka} \right )^2+\frac{2(1-k^3)abc}{(a-kb)(b-kc)(c-kb)}\geq 1$

trong đó $k$ là số thực sao cho $(a-kb)(b-kc)(c-kb)\neq 0$

Bài 7: Cho $a,b,c$ là các số thực đôi một khác nhau. Chứng minh rằng khi đó với mọi $k$, ta luôn có bất đẳng thức:

$\left ( \frac{a+kb}{a-b} \right )^2+\left ( \frac{b+kc}{b-c} \right )^2+\left ( \frac{c+ka}{c-a} \right )^2\geq k^2+1$

Spoiler

Và một kết quả khá nổi tiếng của Vasile Cirtoaje:

Bài 8: Cho $x,y,z$ là các số thực dương thỏa $xyz=1$. Chứng minh rằng:

$\sum \frac{1}{x^2+x+1}\geq 1$




#643560 Số cách để quân cờ không đi qua được bảng

Đã gửi bởi the unknown on 04-07-2016 - 08:34 trong Tổ hợp và rời rạc

Trên bàn cờ vua, một quân Limp King là một quân cờ có thể di chuyển qua một ô vuông đến bất kì ô nào ở xung quanh nó trừ hai ô ở phía Đông Bắc và phía Tây Nam ( hình minh họa).

p78fws.png

Trên một bàn cờ vua $8\times 8$, một số ô vuông đã bị phá hủy. Để đi qua bảng, quân cờ Limp King phải bắt đầu đứng tại một ô bất kì nào đó chưa bị phá hủy của cạnh phía Bắc của bàn cờ, và thực hiện các bước di chuyển qua các ô vuông chưa bị phá hủy và đi đến một ô vuông chưa bị phá hủy của cạnh phía Nam của bàn cờ. Có bao nhiêu cách để phá hủy chính xác $32$ ô cờ để Limp King không thể đi qua được bảng.

Nguồn

Có thể tham khảo đề gốc:

Spoiler




#643251 Hỏi một chút về Gặp gỡ Toán học 2016 tại Đồng Nai

Đã gửi bởi the unknown on 02-07-2016 - 11:03 trong Góc giao lưu

Mà theo em được biết thì chỉ có các học sinh từ lớp 11 trở lên mới được tham dự thôi. Do đó nếu trường có chọn thì em nghĩ trường sẽ chọn các anh chị lớp trên còn lớp 10 muốn đi thì phải có sự đồng ý của hiệu trưởng. Chỉ tiếc là năm nay tổ chức ở Đồng Nai thì gần nhà em mà năm nay em mới lên lớp 10  :P .

@ Min Nq: Theo mình thì đi thi có sợ "bị 0 điểm" hay không thì không quan trọng. Dĩ nhiên kiến thức có thể mình sẽ không hiểu vì chưa được học nhưng quan trọng là được tham gia trải nghiệm. Còn "nhục" thì mình nghĩ chẳng ai đánh giá bạn đâu vì bạn cũng mới chỉ lớp 10  ^_^ . Một điều quan trọng là bạn sẽ còn có được kỷ yếu GGTH nữa mà, cái đó cũng quý đấy. Nếu bạn có cuốn đó thì sau này có thể cho mình mượn được không? ^_^ Mình cũng muốn tham khảo  ^_^ .